1
$\begingroup$

This is a simple question from measure theory.

Fix a measurable space $(E,\mathcal{E})$ and a family $(P_i)_{i\in I}$ of probability measures on $(E,\mathcal E)$ ($I$ is any non-empty set). Let $n\geq 2$ be a positive integer.

Is it true that the following statements are equivalent?

  1. There exists a $\sigma$-finite measure $\mu$ on $(E,\mathcal E)$ such that $P_i$ is absolutely continuous with respect to $\mu$, for all $i\in I$;
  2. There exists a $\sigma$-finite measure $\nu$ on $(E^n,\mathcal E^{\otimes n})$ such that $P_i^{\otimes n}$ is absolutely continuous with respect to $\nu$, for all $i\in I$.

In other (statistical) words, is it true that a statistical model corresponding to one observation is dominated if and only if the statistical model corresponding to $n$ iid copies of that one observation is dominated?

$\endgroup$

1 Answer 1

1
$\begingroup$

Yes, this is true.

For 1. $\Longrightarrow$ 2., simply take $\nu = \mu^{\otimes n}$ to get the dominating measure for the product. This is a $\sigma$-finite measure which dominates the product of the statistical model.

For 2. $\Longrightarrow$ 1., assume w.l.o.g. that $\nu$ is a finite measure; given a $\sigma$-finite measure, there always exist a finite measure equivalent to it. Take $\mu = \pi_{1*}\nu$, where $\pi_1\colon E^n\to E$ is the projection onto the first coordinate, and $\pi_{1*}\nu$ is the pushforward measure ($\pi_{1*}\nu(A) := \nu(\pi_1^{-1}(A))$). You can check that this choice of $\mu$ dominates the statistical model, and it is a finite measure since $\nu$ is. The reason we took $\nu$ to be finite is because otherwise $\nu$ being $\sigma$-finite is not sufficient to guarantee that its pushforward $\pi_{1*}\nu$ is $\sigma$-finite.

$\endgroup$
3
  • $\begingroup$ Hello, thank you! For 1. $\rightarrow$ 2., I agree, the proof is straightforward. For 2. $\rightarrow$ 1. though, it is not as simple. When $\nu$ is finite (hence, wlog, it is a probability measure), simply take its marginals. However, if $\nu$ is not finite, the marginals (push-forwards by the projections) are not $\sigma$-finite, so the proof does not carry on. So, I do not agree with your « w.l.o.g. », it is still unclear whether 2. implies 1. when $\nu$ is not finite. $\endgroup$ Commented Nov 25 at 8:24
  • $\begingroup$ I agree that this case requires care, let me clarify what I meant by "w.l.o.g. that 𝜈 is a finite measure; given a 𝜎-finite measure, there always exist a finite measure equivalent to it". If $\nu$ is not finite, we can just take a finite $\tilde \nu$ which is equivalent to $\nu$ (see en.wikipedia.org/wiki/Σ-finite_measure "Equivalence to a probability measure"). Then we can choose $\mu = \pi_{1*}\tilde \nu$. $\endgroup$ Commented Nov 25 at 8:30
  • $\begingroup$ Ooh that's a super nice fact to know, thank you! $\endgroup$ Commented Nov 25 at 8:43

You must log in to answer this question.

Start asking to get answers

Find the answer to your question by asking.

Ask question

Explore related questions

See similar questions with these tags.